Answer:
4/5
Step-by-step explanation:
Which single transformation can be applied to the graph of y = 72 StartRoot x EndRoot to produce the graph of y = 9 StartRoot x EndRoot?
- vertical translation
- vertical compression
- horizontal translation
- horizontal compression
Answer:
vertical....
Step-by-step explanation:
NawfSide 38 Baby
Answer:
B vertical compression
Step-by-step explanation:
edge :)
A line passes through the points (2, -2) and (8, 1).
Click “show your work” and provide work for calculating the slope and y-intercept. Then, write the equation for the line in slope-intercept form. Please write your answers on the lines provided on the whiteboard.
Answer:
Slope: 1/2Y-intercept: -3Equation: y = 1/2x -3Step-by-step explanation:
You want the slope, y-intercept, and equation for a line that passes through the points (2, -2) and (8, 1).
SlopeThe slope formula is used to find the slope:
m = (y2 -y1)/(x2 -x1)
m = (1 -(-2))/(8 -2) = 3/6 = 1/2
Y-interceptThe slope-intercept equation can be rearranged to give the y-intercept:
b = y - mx
b = 1 -(1/2)(8) = 1 -4 = -3 . . . . . . using (x, y) = (8, 1)
Slope-Intercept FormThe slope-intercept form equation is ...
y = mx +b . . . . . . where m is the slope, and b is the y-intercept
Using the found values, the equation is ...
y = 1/2x -3
SummarySlope: 1/2Y-intercept: -3Equation: y = 1/2x -3y
4
Solve for y.
Then, find the side lengths of the
largest triangle.
Fill in the green blank.
8
X
2
+
2
8
y
y
[?]
X
Enter
Help
Skip
Answer:
y = 4\(\sqrt{5}\) , ? = 10
Step-by-step explanation:
using Pythagoras' identity on the smallest right triangle
x² = 4² + 2² = 16 + 4 = 20 ( take square root of both sides )
x = \(\sqrt{20}\) = \(\sqrt{4(5)}\) = \(\sqrt{4}\) × \(\sqrt{5}\) = 2\(\sqrt{5}\)
using Pythagoras' identity on the middle right triangle
y² = 8² + 4² = 64 + 16 = 80 ( take square root of both sides )
y = \(\sqrt{80}\) = \(\sqrt{16(5)}\) = \(\sqrt{16}\) × \(\sqrt{5}\) = 4\(\sqrt{5}\)
using Pythagoras' identity on the largest right triangle
?² = x² + y² = (2\(\sqrt{5}\) )² + (4\(\sqrt{5}\) )² = 20 + 80 = 100
Take square root of both sides
? = \(\sqrt{100}\) = 10
someone help tyyyyyy
Can someone please help me
Step-by-step explanation:
For y=g(x) and y = f(x), we can set f(x) = g(x) and go from there.
\(4(2)^{-x} +3 = 8-2^{x}\\4 (2)^{-x} - 5 = -2^{x}\\\)
Here, we simply subtracted 8 from both sides
Next, we can substitute u in for 2^x to get
4 * (1/u) - 5 = -u
4/u - 5 = -u
Multiply both sides by u
4 - 5u = -u²
Add u² to both sides
u² -5u +4 = 0
Factor
(u-4)(u-1) = 0
We now know that our solutions to this are u = 4 and u = 1, so we must figure out when 2^x = 4 and 2^x = 1. We can then figure out that 2² = 4 and 2^0 = 1, so our answers are 2 and 0 for the second question.
For the first, we can simply plug 2 and 0 into f(x) to get (2,4) and (2,7) as our answers
Please see screenshot
The graph of the feasible region is attached
How to determine the graph of the feasible regionFrom the question, we have the following parameters that can be used in our computation:
\(\left\{ \begin{array}{lr} y + 7x \ge 10 \\ 8y + 2x \ge 20 \\ y + x \ge 4 \\ y + x\le 10 \\ x \ge 0 \\ y \ge 0\end{array}\)
To plot the graph of the feasible region, we plot each inequality in the domain x ≥ 0 and y ≥ 0
Using the above as a guide, the graph is attached
Read more about feasible region at
https://brainly.com/question/29084868
#SPJ1
12. After 2 hours, Morgan had earned $12.75 at her lemonade stand. After 5
hours, she had earned $44.25. Assuming a linear function, write an
equation in the form y=mx+b that shows the profit earned from selling
lemonade for x hours.
Answer:
y = 10.5x - 8.25
Step-by-step explanation:
m = (44.25 - 12.75)/(5 - 2) = 10.5
y = mx + b
y = 10.5x + b
12.75 = 10.5 × 2 + b
b = -8.25
y = 10.5x - 8.25
A DVD player cost rupees. 3895. What is the cost 45 such players?
Answer:
175,275 rupees
cos xº [Hint: Change degree into radian] find the derivative from definition
The derivative of the function cos(xº) in radians is y' = -sin(xπ/180)
Finding the derivative of the functionFrom the question, we have the following function definition that can be used in our computation:
cos(xº)
Changing the degree into radian, we have
cos(xπ/180)
Express as a function
So, we have
y = cos(xπ/180)
When the cosine function is differentiated, we have
y' = -sin(xπ/180)
Hence, the differentiated function is y' = -sin(xπ/180)
Read more about derivative at
https://brainly.com/question/5313449
#SPJ1
The cost of a newspaper subscription
includes a discounted initial week. Beatriz
pays $55 for 7 weeks of the newspaper
subscription and $100 for 12 weeks.
As a result, the original discounted price is $1 while the weekly normal price is $9.
How does arithmetic work with discounts?The fundamental formula for calculating a reduction is to increase the initial price by the provided percentage rate in decimal form. We must deduct the reduction from the initial price to determine the item's sale price.
From the given information, we have:
d + 6x = 55 (discounted price for 7 weeks)
d + 11x = 100 (discounted price for 12 weeks)
We can solve this system of equations by eliminating d. Subtracting the first equation from the second, we get:
5x = 45
Dividing both sides by 5, we get:
x = 9
Substituting x = 9 into one of the equations, we can solve for d. Using the first equation, we get:
d + 6(9) = 55
d + 54 = 55
d = 1
To know more about Discount visit:
https://brainly.com/question/1548141
#SPJ1
Find the equation of a line with a slope of 13/2
that passes through the point (−2, — 10).
Answer: \(y=\frac{13}{2} x+3\)
Step-by-step explanation:
Remember the point-slope equation which is \(y-y_{1} = m(x-x_{1} )\) where\((x_{1} , y_{1} )\) is your point and \(m\) is your slope.
Given that we substitute what you have:
\(y-(-10) =\frac{13}{2} (x - (-2))\)
Minus and minus give us positive:
\(y+10 =\frac{13}{2} (x +2)\)
Multiply slope into the parenthesis:
\(y+10= \frac{13}{2}x + \frac{13}{2}*2\\\)
Calculate it:
\(y+10= \frac{13}{2}x +13\)
Isolate the \(y\) by subtracting 10 from both sides:
\(y=\frac{13}{2} x + 13 -10\)
Your final equation is:
\(y=\frac{13}{2} x +3\)
Hope this makes sense!
And here's the graph to prove that the line actually goes through the point \((-2,-10)\):
find the volume of the rectangular prism.2/3 mm2/3mm1 3/4 mm
Find the volume of the rectangular prism
\(\begin{gathered} \text{Volume =Length}\times width\times height \\ V=\text{lwh} \end{gathered}\)where L = 2/3mm
width w = 2/3mm
height = 1 3/4 = 7/4mm
\(undefined\)if 6 people can walk 12 dogs, how many people are needed to walk 24 dogs
Answer:
12 people are need to walk 24 dogs
Step-by-step explanation:
12/2=6
24/2=12
Answer:
12 people can walk 24 dogs.
Step-by-step explanation:
Given information,
→ 6 people can walk 12 dogs.
→ People required for 24 dogs = ?
Let us assume that,
→ Missing value = p
Forming the equation,
→ 6/12 = p/24
Then the value of p will be,
→ 6/12 = p/24
→ p/24 = 6/12
→ p × 12 = 6 × 24
→ 12p = 144
→ p = 144/12
→ [ p = 12 ]
Hence, the value of p is 12.
QUESTION IN PICTURE
Please explain your answer in steps, thank you.
We can complete the blanks with the following ratios:
(7.5 mi/1) * (1 mi/ 5280 ft) * (400ft/1 yd) * (3 ft/1 ft) =33 flags
Since we do not need a flag at the starting line, then 32 flags will be required in total.
How to obtain the number of flagsTo solve the problem, we would first convert 400 yds to feet and miles.
To convert to feet, we multiply by 3. This gives us: 400 yd * 3 = 1200 feet.
To convert to miles, we would have 0.227 miles.
Now, we divide the entire race distance by the number of miles divisions.
This gives us:
7.5 mi /0.227 mi
= 33 flags
Learn more about miles-to-yards conversion here:
https://brainly.com/question/1609225
#SPJ1
if (2i/2+i) - 3i(3+i) = a + bi then a= ____ and b=_____
A = 1/10, -10, 1/50, -1/10
B = i/10, -10i, -1/10, -1/50
The value of a and b in the given complex expression is 1/10 and -1/10 respectively.
This is a problem related to the complex numbers. The complex numbers has a general form of (a+bi) where 'i' is the imaginary number or √-1. The part without an 'i' is called Real Part and the part with an 'i' is called Imaginary Part.
(2i/2+i) - 3i/(3+i) = a + bi
{ 2i(3+i) - 3i(2+i) }/ (2 + i)(3 + i) = a+bi
6i + 2i² - 6i - 3i² / (2 + i)(3 + i) = a+bi
(-2 + 3) / (6 + 5i - 1) = s+bi
1 / (5 + 5i) = a+bi
Now we multiply top and bottom by 5 - 5i :
5 - 5i / (5 + 5i)(5 - 5i) = a+bi
5 - 5i / 25 -25i² = a+bi
5 - 5i / 50 = a+bi
1/10 - 1/10i = a+bi
On comparing the real and imaginary part on the both sides:
a= 1/10 , b= -1/10.
To know more about the complex numbers, refer:
https://brainly.com/question/20566728?referrer=searchResults
When x = 4, what is 6x – 18?
Hey there!☺
\(Answer:\boxed{6}\)
\(Explanation:\)
If x is equal to 4, then you will have to multiply 6 by x which is 4.
\(6*4=24\)
Now subtract 18 from 24.
\(24-18=6\)
\(6x-18=6\)
6 is your answer.
Hope this helps!☺
Find the quotient of z₁ by z2. Express your answer in
trigonometric
form.
² - 3 (0 (4) + (*))
Z₁ cos
+/sin
Z₂
²2 = 7 (cos(377)+
COS
8
O A. 7 (cos (577) + i sin (5/77))
8
B.
21(cos(577)+isin (577))
8
OC. 21 cos
21(cos(-7)+ i sin(-77))
O D. 7 (cos(-7) + + sin(-7))
i
+/sin
37T
8
The quotient of z₁ by z₂ in trigonometric form is:
7/21 * (cos(584°) + i sin(584°))
To find the quotient of z₁ by z₂ in trigonometric form, we'll express both complex numbers in trigonometric form and then divide them.
Let's represent z₁ in trigonometric form as z₁ = r₁(cosθ₁ + isinθ₁), where r₁ is the magnitude of z₁ and θ₁ is the argument of z₁.
We have:
z₁ = 7(cos(577°) + i sin(577°))
Now, let's represent z₂ in trigonometric form as z₂ = r₂(cosθ₂ + isinθ₂), where r₂ is the magnitude of z₂ and θ₂ is the argument of z₂.
From the given information, we have:
z₂ = 21(cos(-7°) + i sin(-77°))
To find the quotient, we divide z₁ by z₂:
z₁ / z₂ = (r₁/r₂) * [cos(θ₁ - θ₂) + i sin(θ₁ - θ₂)]
Substituting the given values, we have:
z₁ / z₂ = (7/21) * [cos(577° - (-7°)) + i sin(577° - (-7°))]
= (7/21) * [cos(584°) + i sin(584°)]
The quotient of z₁ by z₂ in trigonometric form is:
7/21 * (cos(584°) + i sin(584°))
Option C, 21(cos(-7°) + i sin(-77°)), is not the correct answer as it does not represent the quotient of z₁ by z₂.
For more questions on trigonometric form
https://brainly.com/question/31744474
#SPJ8
In the diagram below, the line of sight from the park ranger station, P, to the lifeguard chair, L, on the beach of a lake is perpendicular to the path joining the campground, C, & the first aid station, F. The campground is 0.35 mile from the lifeguard chair. The straight paths from both the campground and first aid station to the park ranger station are perpendicular.
If the path from the park ranger station to the campground is 0.65 mile, determine and state, to the nearest
hundredth of a mile,
a. Find the length of PL
Whwn the line of sight from the park ranger station, P, to the lifeguard chair, L, the length of PL is 0.76 miles.
How to calculate the valueIt should be noted that since the line of sight from the park ranger station, P, to the lifeguard chair, L, on the beach of a lake is perpendicular to the path joining the campground, C, and the first aid station, F, then the path from the park ranger station to the lifeguard chair is the hypotenuse of a right triangle with legs of 0.35 miles and 0.65 miles.
The Pythagorean theorem states that in a right triangle, the square of the hypotenuse is equal to the sum of the squares of the legs. Therefore, the length of PL is equal to:
= ✓(0.35² + 0.65²)
= 0.76 miles.
Learn more about Pythagorean on
https://brainly.com/question/343682
#SPJ1
Question 2: Leilamade a one-time investment of $12 000.00 in a
registered retirement savings plan (RRSP) at 2.65%, compounded
semi-annually. She plans to withdraw the money when she retires in 30
years.
a) Determine the value of the investment when she retires. Show your
work.
I
b) Calculate the rate of return [note:] over the 30 years. Show your work.
the value of the investment when she retires is $26434.92.
What is the compound interest?Compound interest is when you earn interest on both the money you've saved and the interest you earn.
Formula:
A = P(1 + {r}/{n})^{n.t}
A = final amount
P = initial principal balance
r = interest rate
n = number of times interest applied per time period
t = number of time periods elapsed
here, we have,
given that,
Leilamade a one-time investment of $12 000.00 in a
registered retirement savings plan (RRSP) at 2.65%, compounded
semi-annually.
She plans to withdraw the money when she retires in 30
years.
So, she get finally is,
using the formula we get,
$26434.92
hence, the value of the investment when she retires is $26434.92.
To learn more on Compound interest click:
brainly.com/question/29335425
#SPJ1
2. A coffee maker and a juice maker are making beverages. The rate at
which the coffee is being made is given by y = 6x, where y represents the
amount of coffee in mL and x represents the time passed in seconds. The
amount of juice that has been made at different times is summarized in
the table.
thing Company
A. At what rate is the coffee maker making coffee?
B. At what rate is the juice maker making juice?
C. Is juice or coffee being made at a faster rate?
Time (s)
0
1
2
3
Juice (mL)
0
8
16
24
(A) The rate of making coffee by coffee maker is 6 mL/second.
(B) The rate of making juice by juice maker is 8 mL/second.
(C) Juice is being made at a faster rate.
What is ratio?Ratio basically compares quantities, that means it show value of one quantity with respect to other quantity.
If a and b are two values, their ratio will be a:b,
Given that,
The rate of coffee made by coffee maker is represented by,
y = 6x (1)
Here, y represents the amount of coffee in mL and x represents the time passed in seconds.
Also, The rate of juice made by juice make can be represented by the given table as,
k = 8h (2)
Here, k represents the amount of juice in mL and h represents the time passed in seconds.
(A)
To find the rate of making coffee, solve the equation (1),
y = 6x
⇒ y/x = 6 mL/second
The rate of making coffee by coffee maker is 6 mL/second.
(B)
To find the rate of making juice, solve the equation (2),
k = 8h
⇒ k/h = 8 mL/second
The rate of making juice by juice maker is 8 mL/second.
(C)
Since, the juice is made at the rate of 8 mL/second and the coffee is made at the rate of 6 mL/second.
Therefore, juice is being made at a faster rate.
To know more about Ratio on:
https://brainly.com/question/23724140
#SPJ1
please help!!!!!!!!!!!!!
Answer:
It is 5x - 9 = 21
Step-by-step explanation:
OOF DELTA MAtH suCKS
Simplify the given algebraic expression.
9{9b-[2-3(9b-5)]}
Please I need an answer for this
Thanks!!!
After simplifying the algebraic expression 9 { 9 b - [ 2 - 3 ( 9 b - 5 ) ] }, we get the result as 324 b - 153.
We are given the algebraic expression:
9 { 9 b - [ 2 - 3 ( 9 b - 5 ) ] }
We need to simplify the expression.
For this, we will use the BODMAS rule.
B - Brackets
O - Of
D - Division
M - Multiplication
A - Addition
S - Subtraction
So, simplifying the expression, we get that:
9 { 9 b - [ 2 - 3 ( 9 b - 5 ) ] }
= 9 { 9 b - [ 2 - 27 b + 15 ] }
= 9 { 9 b - 2 + 27 b - 15 }
= 9 { 9 b - 2 + 27 b - 15 }
= 81 b - 18 + 243 b - 135
= 324 b - 153
Therefore, after simplifying the algebraic expression 9 { 9 b - [ 2 - 3 ( 9 b - 5 ) ] }, we get the result as 324 b - 153.
Learn more about algebraic expression here:
https://brainly.com/question/4344214
#SPJ9
Identify the coordinates of the point in polar form based upon the given conditions.
Use pi for , or click the yellow arrow for the MathQuill.
Thus, the coordinates of R in rectangular form are (x, y) = (-5/√2, 5/√2).
To identify the coordinates of the point in polar form based on the given conditions, we need to use the following polar coordinate system:In the polar coordinate system, a point is located by its distance from the origin and the angle it makes with the positive x-axis, also known as polar angle.
The distance from the origin is denoted by r and the polar angle is denoted by θ. The coordinates of a point in polar form are written as (r, θ).Here are some examples of how to identify the coordinates of a point in polar form:Example 1:
Find the coordinates of the point P with rectangular coordinates (2, 3).Solution: We can use the following formulas to find the polar coordinates: r = √(x² + y²) and θ = tan⁻¹(y/x). Therefore, we have:
r = √(2² + 3²) = √13θ = tan⁻¹(3/2)
Thus, the coordinates of P in polar form are
(r, θ) = (√13, tan⁻¹(3/2)).
Example 2: Find the coordinates of the point Q with rectangular coordinates (-4, -4).Solution: We can use the following formulas to find the polar coordinates:
r = √(x² + y²)
and
θ = tan⁻¹(y/x).
Therefore, we have:r =
√((-4)² + (-4)²) = 4√2θ = tan⁻¹((-4)/(-4)) = tan⁻¹(1) = π/4
Thus, the coordinates of Q in polar form are (r, θ) = (4√2, π/4).Example 3: Find the coordinates of the point R with polar coordinates (5, 3π/4).Solution: We can use the following formulas to find the rectangular coordinates: x = r cos θ and y = r sin θ. Therefore, we have:x
= 5 cos(3π/4) = -5/√2y = 5 sin(3π/4) = 5/√2
For such more question on coordinates
https://brainly.com/question/31293074
#SPJ8
When converted to a household measurement, 9 kilograms is approximately equal to a) 9000 grams. b) 9000 ounces. c) 19.8 ounces. d) 19.8 pounds.
Answer:
a) 9000 gam
Step-by-step explanation:
Answer:
a) 9000 grams
Step-by-step explanation:
1 kg=1000 g
9*1000=9000
(5) La tabla muestra una proporción utilizada para hacer pintura naranja. Usando la tabla, ¿cuántos cuartos de galón de pintura amarilla se necesitarian para hacer este tono de naranja si usara 9 cuartos de galón de pintura roja?
Como no tenemos tabla, vamos a utilizar una tabla propia:
Número de 1/4 gal Pintura amarilla: 0 1 2 3 4
Número de 1/4 gal Pintura roja: 0 2 4 6 8
Para calcular cuantos cuartos de galón de pintura amarilla se necesitan para hacer este tono de naranja si se utilizan 9 cuartos de galón de pintura roja usaremos la Regla de Tres:
Si por cada 4 cuartos de galón de pintura roja necesitamos 2 cuartos de galón de pintura amarilla, para 9 cuartos de galón de pintura roja, necesitamos:
\(9\cdot(\frac{2}{4})=9\cdot0.5=4.5\)Necesitamos 4.5 cuartos de galón de pintura amarilla.
CS Algebra
Test Iten
Given the equations below, what is the product of x and y?
3
8(x - 2) = 2x + 8
ay+9 = -2(y + 1)
=
A 16
B 4
C4
0 c
D -16
Answer:
Hope this helps :)
Step-by-step explanation:
8(x - 2) = 2x + 8
y+9 = -2(y + 1)
value of x in 8(x - 2) = 2x + 8
x=4
substitue
y+9=−2(y+1)
value of y y+9=−2(y+1)
y= - 11/3 or 3.66...
x=4
y=4 (I rounded 3.66)
Please help ASAP!! I mark brainliest.
Answer:
56
Step-by-step explanation:
8 / 5 = 1.6
72 / 45 = 1.6
35 x 1.6 = 56
*check* 56 / 35 = 1.6
Helppppp What is the volume?
Answer:
Step-by-step explanation:
The volume is found by multiplying all the sides of the rectangular shape.
Thus the volume = 6 * 8 * 10 = 480 cubic feet
Hope that helps!
Answer:
480 cubic feet
Step-by-step explanation:
The volume of any prism is Bh, where B = AREA of the base and h = height of prism.
We're given:
Dimensions of base as 6 and 8Height as 10Now, we solve the volume.
\((6)(8)(10)\) \((48)(10)\) \(480\)Therefore, the answer is 480 cubic feet.
What is the solution to |x-2| + 3 > 17?
Ox<-12 or x > 16
Ox<-14 or x>7
O-12
O-14
Answer:
x<-12 or x>16
Step-by-step explanation:
Answer:
|x - 2| + 3 > 17
|x - 2| > 14
x - 2 < -14 or x - 2 > 14
x < -12 or x > 16
Solve for x.
0.5x – 0.3 = 0.3
4
x = [?]
Answer:
0.5x - 0.3 = 0.3
0.5x = 0.3 + 0.3
0.5x = 0.06
x = 0.06/0.5
x = 0.12
here is the value of x